Q17

 
b91302310
Thanks Received: 13
Atticus Finch
Atticus Finch
 
Posts: 153
Joined: August 30th, 2010
 
 
 

PT19,S3,Q17-As presented in the passage

by b91302310 Mon Dec 13, 2010 5:16 am

There are two possible answer choices in my mind, which are A and E.

For A, I think the wording "increase significantly" is too strong, otherwise it is really nice to be a correct answer.

For E, I think this one is not good enough because it treats the time-theory as to predict the weather in the future but not the relationship between weather and varities of species. However, although not touching on the core issue, I think this one at least not so much strong as A.

So, is there any reason for why A is better than E?

Thanks.
User avatar
 
bbirdwell
Thanks Received: 864
Atticus Finch
Atticus Finch
 
Posts: 803
Joined: April 16th, 2009
 
This post thanked 1 time.
 
 

Re: PT19,S3,Q17-As presented in the passage

by bbirdwell Tue Dec 14, 2010 12:44 am

So, the real question is this: what do we know about the time-theory?

And the answer is this: there are more species in the tropics because species there have had more time to develop than species in other places, because other places (temperate and arctic zones) have been interrupted by ice ages.

Put simply: ice ages in non-tropical places keep the number of species low.

Thus, (A) is very well supported -- without the ice ages, the number of species would have increased. "Significantly" simply means "enough to notice."

You're right about (E) - nothing is said about the future.
I host free online workshop/Q&A sessions called Zen and the Art of LSAT. You can find upcoming dates here: http://www.manhattanlsat.com/zen-and-the-art.cfm
 
bigtree65
Thanks Received: 2
Forum Guests
 
Posts: 38
Joined: September 16th, 2011
 
 
 

Re: Q17

by bigtree65 Fri Dec 09, 2011 7:00 pm

But the author goes on to say that arctic regions are unaffected by ice ages. Wouldn't this mean that whether or not there are ice ages would not affect the amount of species in high latitudes? Or am I making a mistaken assumption by thinking that high latitudes are equivalent to arctic regions?
 
lhermary
Thanks Received: 10
Atticus Finch
Atticus Finch
 
Posts: 160
Joined: April 09th, 2011
 
 
 

Re: Q17

by lhermary Tue Jan 10, 2012 3:21 pm

Why is C wrong?
User avatar
 
bbirdwell
Thanks Received: 864
Atticus Finch
Atticus Finch
 
Posts: 803
Joined: April 16th, 2009
 
This post thanked 1 time.
 
 

Re: Q17

by bbirdwell Wed Jan 18, 2012 1:55 pm

Regarding (C):
Time-theory doesn't state that "in the absence of ice ages, all climate zones will resemble one another." Time-theory is not even really about the climate zones at all -- it's about the number of species in those zones.
I host free online workshop/Q&A sessions called Zen and the Art of LSAT. You can find upcoming dates here: http://www.manhattanlsat.com/zen-and-the-art.cfm
 
goriano
Thanks Received: 12
Atticus Finch
Atticus Finch
 
Posts: 113
Joined: December 03rd, 2011
 
 
 

Re: Q17

by goriano Sat Jul 14, 2012 2:32 pm

bigtree65 Wrote:But the author goes on to say that arctic regions are unaffected by ice ages. Wouldn't this mean that whether or not there are ice ages would not affect the amount of species in high latitudes? Or am I making a mistaken assumption by thinking that high latitudes are equivalent to arctic regions?


I think the problem with your reasoning is that you are incorporating the author's analysis of the time-theory into evaluating the answer choices. The question stem asks you to answer based on the PRINCIPLES of the time theory, which would presumably do not include the author's viewpoints as they are intended to discredit the theory.
 
john.o.wray
Thanks Received: 5
Vinny Gambini
Vinny Gambini
 
Posts: 13
Joined: February 08th, 2012
 
 
 

Re: Q17

by john.o.wray Tue Jul 17, 2012 11:12 pm

I had it down to A and C and spent a good amount of time deciding between the two.

I concluded that C was wrong because it just goes one step farther than A and it's not supported. Also, if you read it closely, it says that the climate will resemble the tropics. How would the time theory even support that, since it's talking about rate of species growth?

E is wrong because it contradicts lines 14-15 where it says that arctic conditions have not been affected by ice ages. So why would a new one affect it?
 
hsu.chiang
Thanks Received: 0
Vinny Gambini
Vinny Gambini
 
Posts: 2
Joined: August 21st, 2011
 
 
 

Re: Q17

by hsu.chiang Sun Jul 14, 2013 1:28 pm

Could anyone help me with D?
Why D is wrong?
 
kthalen
Thanks Received: 0
Vinny Gambini
Vinny Gambini
 
Posts: 4
Joined: May 23rd, 2013
 
 
 

Re: Q17

by kthalen Thu Aug 01, 2013 2:01 pm

I was also unsure why D is wrong, I narrowed it down to A and D but do not see why D is not correct.
User avatar
 
pridefc
Thanks Received: 0
Vinny Gambini
Vinny Gambini
 
Posts: 3
Joined: April 14th, 2013
 
 
 

Re: Q17

by pridefc Mon Aug 26, 2013 2:46 pm

kthalen Wrote:I was also unsure why D is wrong, I narrowed it down to A and D but do not see why D is not correct.


I believe that based on Time Theory's principles, it gives an explanation as to why the gradient is present (in part due to ice ages). The passage does not state that there has been a current ice age or an expectation for one in the near future. So species in the arctic will have more time to emerge uninterrupted and could out pace the rate in which new species emerge when compared to the tropics.

So this cannot be most strongly supported with that doubt in place.
 
StratosM31
Thanks Received: 0
Jackie Chiles
Jackie Chiles
 
Posts: 31
Joined: January 03rd, 2020
 
 
 

Re: Q17

by StratosM31 Wed Jun 17, 2020 4:40 pm

john.o.wray Wrote:E is wrong because it contradicts lines 14-15 where it says that arctic conditions have not been affected by ice ages. So why would a new one affect it?


Careful, that's an opposition stated by the author towards the time theory, not what the time-theory itself is stating!

E is tricky because it is an opposition to the author's critique on the theory, but it does not necessarily mean that the theory itself predicts it. It might simply be the case that the theory did not take it into account.

Before Einstein came up with relativity theory, physicists only worked with the tools of classical physics that were at their disposal. This does not imply, thouugh, that they believed that a standing object has no velocity independent on the reference system. Probably they did not even think about this problem, or they thought that it was so negligible that it could not affect the universal correctness of classical physics...